For any two different real numbers $x$ and $y$, we define $D(x,y)$ to be the unique integer $d$ satisfying $2^d\le |x-y| < 2^{d+1}$. Given a set of reals $\mathcal F$, and an element $x\in \mathcal F$, we say that the scales of $x$ in $\mathcal F$ are the values of $D(x,y)$ for $y\in\mathcal F$ with $x\neq y$. Let $k$ be a given positive integer. Suppose that each member $x$ of $\mathcal F$ has at most $k$ different scales in $\mathcal F$ (note that these scales may depend on $x$). What is the maximum possible size of $\mathcal F$?
Problem
Source: ISL 2019 C9
Tags: combinatorics, IMO Shortlist, IMO Shortlist 2019, orz, cOPtorics
23.09.2020 02:52
Fantastic problem, well worth extending the C shortlist to a 9th problem for.
23.09.2020 04:20
This is a difficult one with an extremely simple solution. It took me over 5 hrs to figure out a solution that is probably only 5 lines long lol. Therefore I'd like to talk about how I found the right approach (I took a really long detour though).
23.09.2020 12:21
Apparently everyone is doing this with the lemma. My solution is fairly similar, although it's not the same, at least cosmetically. Here's the gist. By taking a dilation of factor $1+\varepsilon$ for suitable small $\varepsilon$, one can ensure that every number has an interval in which to move, without changing any scales. Suppose that $2^d$ is the largest scale that exists in our set; divide the set into $A, B, C$ where the distance from anything in $A$ to anything in $C$ is at least $2^d$. Delete $C$, duplicate $A$, and place the new copy of $A$ in the same position except perturbed very slightly to the right (by say $2^{n_1}$ where $n_1$ is very negative). Repeat this until none of the original scales exist, i.e. all points are duplicates (of duplicates of ...) of the same point. It's easy to see that the number of scales is still at most $k$ for all points, as long as the perturbation is small enough. This process never decreases the number of points, but since we can choose the perturbation amount, we can essentially wlog that the final set can be split into two sets which all have the same scale to each other (which can be further split etc etc), forming a cantor-set like construction which obviously has $2^k$ points.
23.09.2020 16:38
While everyone is railing on the PSC for IMO 2020, I'd like to point out that this problem was lowest-rated quality-wise by the IMO 2019 Jury.
23.09.2020 17:49
CantonMathGuy wrote: While everyone is railing on the PSC for IMO 2020, I'd like to point out that this problem was lowest-rated quality-wise by the IMO 2019 Jury. I think this problem might be too hard for IMO, but the quality of this problem is definitely the best (or maybe second to C6 since I haven't taken a careful look at that problem). I'd also like to hop on the roasting train and mention how the difficulties of C7, C8 and N8 are so not matching to their numbers
27.09.2020 16:45
The problem is all about proving that for any finite set $A$ the following holds:$$\sum_{x\in A} 2^{-scales(x)} \le 1$$. Once you notice that the problem is no longer hard. We induct on $|A|$ (for $|A|=1,2$ it is obvious). Let $a_1=minA$ the minimum element of $A$ and $a_2=maxA$ and let $2^c\le a_2-a_1 <2^{c+1}$. The interval $[a_1,a_2]$ is made up of three subintervals : $[a_1,a_2-2^c] \to A_1$ $(a_2-2^c,a_1+2^c)\to M$ $[a_1+2^c,a_2]\to A_2$ and let $A_1\cup M\cup A_2 =A $ be their corresponding intersections with the set $A$. Both $A_1,A_2$ are nonempty , the induction hypothesis gives $$\sum_{x_1\in A_1\cup M} 2^{-scales(x_1)} \le 1, \sum_{x_2\in A_2\cup M} 2^{-scales(x_2)} \le 1$$Every $x_1\in A_1 $ and $x_2\in A_2$ do not have the scale $c$ in $A_1\cup M$ and in $A_2\cup M$ respectively but in $A$ they do (and thus their scales are increased by $1$ in the superset$A$ , $D(x_1,a_2)=D(x_2,a_1)=c$) Every $m\in M$ has at least as many scales in $A$ as it has in either $A_1\cup M,A_2\cup M$ and we conclude that $$\sum_{x\in A} 2^{-scales(x)} \le \frac{\sum_{x_1\in A_1\cup M} 2^{-scales(x_1)}+\sum_{x_2\in A_2\cup M} 2^{-scales(x_2)}}{2}\le 1$$. We get that $|\mathcal F |\le 2^k $ and the construction is $\mathcal F=(1,2,...,2^k)$.
05.01.2023 01:40
We claim the answer $2^k.$ Clearly $\mathcal F =\left \{ 1,2,\dots ,2^k\right \}$ satisfies as $\forall x,y\in \mathcal F :0\leq D(x,y)< k$. In order to prove the bound $|\mathcal F|\leq 2^k$ we establish the following sufficent Lemma. For any finite set $A\subset \mathbb R$ define $s(x)$ as a number of different scales of $x\in A$ in $A$. Then $$\sum_{x\in A} 2^{-s(x)} \leq 1.$$Proof. Induct on $n=|A|$. Base case $n=1$ is trivial since $LHS=1$. Now assume $A=\left \{ x_1,x_2,\dots ,x_n\right \}$ with $x_i<x_{i+1}.$ Let $d=D(x_1,x_n)$ denotes the maximal scale in $A$ and define $P,Q,R\subset A$ as $$P=\left \{ x\in A\mid D(x_1,x)=d\right \} \quad Q=\left \{ x\in A\mid D(x,x_n)=d\right \} \quad R=A\backslash (P\cup Q).$$Observe that $P\cap Q=\emptyset$ since otherwise $$\forall x\in P\cap Q:x_n-x_1=(x_n-x)+(x-x_1)\geq 2^d+2^d=2^{d+1}.$$Obviously $D(x,y)\neq d$ for either $x\in P,$ $y\in P\cap R$ or $x\in Q,$ $y\in Q\cap R,$ thus by the inductive hypothesis on $P\cup R,\text{ }Q\cup R$ we easily complete the inductive step with $$2\sum_{x\in A} 2^{-s(x)}=\left( 2\sum_{x\in P} 2^{-s(x)} +\sum_{x\in R} 2^{-s(x)}\right) +\left( 2\sum_{x\in Q} 2^{-s(x)} +\sum_{x\in R} 2^{-s(x)}\right) \leq 1+1 =2\text{ } \Box$$